Asymptotique de $\sum_{k=1}^{[nx]}k^{-a}(n-k)^{-b}$

P.2
P.2
Modifié (November 2022) dans Analyse
Dans la formule du titre  $\quad\displaystyle \sum_{k=1}^{[nx]}\frac{1}{k^{a}(n-k)^{b}},\quad$ je suis intéressé par le cas $a>1,\ b>1,\ 0<x<1,$
et une estimation asymptotique de l'expression pour $n\to \infty.$ Ici $[nx]$ veut dire partie entière.

Réponses

  • Cas particulier $a=b=s>1$ 
    Pour $s>1$, il me semble avoir déjà vu que $\sum_{k=1}^{n-1} \frac{1}{k^s(n-k)^s} \sim \frac{2\zeta(s)}{n^s}$ 
  • On pourrait s'en sortir par comparaison avec une intégrable, non ?
  • En tout cas joli problème et c'est le genre que aime @jandri.
    J'ai copié/collé la formule donnée par etanche dans mon navigateur et j'obtiens ce lien https://artofproblemsolving.com/community/c7h2232644p17065932

    @JLapin Fonce sur ton idée 
    Le 😄 Farceur


  • Calli
    Modifié (November 2022)
    Bonjour,
    On a \begin{eqnarray*} \left|\sum _{k=1} ^{\lfloor {xn}\rfloor } \frac{1}{k^{a} (n-k)^b } - \frac{1}{n^{b}} \sum _{k=1} ^{\lfloor {xn}\rfloor } \frac{1}{k^{a}} \right| &\leqslant & \sum _{k=1} ^{\lfloor {xn}\rfloor } \frac{1}{k^{a}} \left| \frac{1}{(n-k)^{n} } - \frac{1}{n^{b}} \right|\\ &\underset{\text{IAF}}{\leqslant }& \sum _{k=1} ^{\lfloor {xn}\rfloor } \frac{1}{k^{a}} \frac{bk}{(n-k)^{b+1} } \\ &\leqslant & \sum _{k=1} ^{\lfloor {xn}\rfloor } \frac{1}{k^{a}} \frac{bk}{((1-x)n)^{b+1} } \\ &\leqslant & \frac{b}{((1-x)n)^{b+1} } \left( 1+\int _{1} ^{\lfloor {xn}\rfloor} \frac{\mathrm{d}t}{t^{a-1}}\right) \\ &=& \frac{b}{((1-x)n)^{b+1} } \left(1+ \frac{\lfloor {xn}\rfloor^{2-a} -1 }{2-a} \right)\\ &=& O\!\left( \frac{n^{\max (0,2-a)} }{n^{b+1}} \right) \end{eqnarray*} Donc \[\sum _{k=1} ^{\lfloor {xn}\rfloor } \frac{1}{k^{a} (n-k)^b } = \frac{1}{n^{b}} \sum _{k=1} ^{\lfloor {xn}\rfloor } \frac{1}{k^{a}} + O\!\left( \frac{n^{\max (0,2-a)} }{n^{b+1}} \right) \sim \frac{\zeta (a)}{n^{b}}\] car $b+1-(2-a)=b+a-1>b$.

    Edit : J'ai oublié de traiter le cas $a=2$ à part, cf. remarque de jandri.
  • gebrane
    Modifié (November 2022)
    D'après le résultat de @Calli que je salue, en combinant  ce qu'a dit etanche, on aura $$\lim_{n\to +\infty}   \frac{n^a}{\zeta (a)}\sum _{k=\lfloor {xn}\rfloor } ^{n-1 } \frac{1}{k^{a} (n-k)^a } =1$$ C'est  étrange me semble-t-il.
    Le 😄 Farceur


  • jandri
    Modifié (November 2022)
    Si on change $k$ en $n-k$ dans le résultat obtenu par Calli on obtient $\displaystyle\sum _{k=\lfloor {(1-x)n}\rfloor+1  } ^{n-1}\frac{1}{(n-k)^{a} k^b }  \sim \frac{\zeta (a)}{n^{b}}$.
  • Calli
    Modifié (November 2022)
    Je ne vois pas le problème. On a $\displaystyle \sum _{k=\lfloor {xn}\rfloor }^{n-1} \frac{1}{k^{a} (n-k)^{a} } = \sum _{k=1} ^{\lceil {(1-x)n}\rceil} \frac{1}{k^{a} (n-k)^{a} }$, donc ça m'a l'air cohérent.

    Edit : Je n'avais pas vu la réponse de @jandri
  • Calli
    Dans ta démonstration il faut traiter à part le cas $a=2$ puisque dans ce cas on obtient un $O\!\left( \dfrac{\ln n}{n^{b+1}} \right)$.
  • En effet @jandri, je n'y avais pas fait attention.
Connectez-vous ou Inscrivez-vous pour répondre.